WZH 发表于 2021-7-17 23:56:51

RTX5中消息邮箱出现的坑

问题出现在使用静态分配策略初始化消息邮箱的情况中,产生错误的代码如下:
<blockquote><blockquote>#define DEBUG_MAX_MES<span style="white-space:pre">                </span>(256)<span style="white-space:pre">                        </span>//缓存的消息数量·static __ALIGNED(4) uint8_t Message_Buf;static osRtxMessageQueue_t visual_debug_queue_work_buf __QUEUE;static const osMessageQueueAttr_t        visual_debug_queue_attr={        .cb_mem=&visual_debug_queue_work_buf,        .cb_size=sizeof(visual_debug_queue_work_buf),        .mq_mem=Message_Buf,        .mq_size=sizeof(Message_Buf),        .attr_bits=0,        .name="Visual_Debug_Queue"};static osMessageQueueId_t visual_debug_queue=NULL;//......//消息邮箱初始化bool Debug_Message_Init(void){//....//出现问题的地方        visual_debug_queue=osMessageQueueNew(DEBUG_MAX_MES,sizeof(Visual_Message),&visual_debug_queue_attr);        if(visual_debug_queue==NULL){                return false;        }//....}
乍一看这个初始化代码好像没什么问题,初始化步骤都是按照手册中要求初始化的,附上手册英文原文:data:image/png;base64,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
中文百度翻译版
data:image/png;base64,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
代码里结构体肯定是32bit对齐的,而且内存大小也满足手册上面的要求,但是实际使用时,根本无法初始化成功。
问题定位
通过苦逼的单步调试中,找到了问题所在,见下图
data:image/png;base64,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
在rtx_msgqueue.c的255行-256行中,进行了上图的计算,这里的block_size诡异地加上了sizeof(os_message_t)这个结构体,代码后面有对我们预留的缓冲区大小和这个size进行比较,很显然我们上面的代码中缓冲区偏小,因此无法初始化成功。
将上面的代码缓冲区部分修改为:
static __ALIGNED(4) uint8_t Message_Buf;代码中osRtxMessageQueue_t与os_message_tc是等效的。
经过上述修改后初始化成功了。
总结
注意RTX5消息邮箱初始化与FreeRTOS的区别。

WZH 发表于 2021-7-18 00:00:44

#define DEBUG_MAX_MES                (256)                        //缓存的消息数量·
#define DEBUG_MES_TASK_STK        (512)                        //调试信息栈容量
#define __QUEUE                        __attribute__((section(".bss.os.msgqueue.cb")))
typedef struct{
        uint8_t* mes;
        uint32_t len;
}Visual_Message;

static __ALIGNED(4) uint8_t Message_Buf;
static osRtxMessageQueue_t visual_debug_queue_work_buf __QUEUE;
static const osMessageQueueAttr_t        visual_debug_queue_attr={
        .cb_mem=&visual_debug_queue_work_buf,
        .cb_size=sizeof(visual_debug_queue_work_buf),
        .mq_mem=Message_Buf,
        .mq_size=sizeof(Message_Buf),
        .attr_bits=0,
        .name="Visual_Debug_Queue"
};
static osMessageQueueId_t visual_debug_queue=NULL;
//...
//...
//初始化消息邮箱
bool Visual_Debug_Init(void){
//...
//问题代码处
        visual_debug_queue=osMessageQueueNew(DEBUG_MAX_MES,sizeof(Visual_Message),&visual_debug_queue_attr);
        if(visual_debug_queue==NULL){
                return false;
        }
//...

}

eric2013 发表于 2021-7-19 00:32:31

谢谢楼主分享。

WZH 发表于 2021-7-19 13:30:53

另外更正一下,osRtxMessage_t与os_message_t是等效的,原文写成了osRtxMessageQueue_t与os_message_tc是等效的,抱歉。

WZH 发表于 2021-7-27 21:44:01

也可以通过内置的宏定义osRtxMessageQueueMemSize计算所需内存大小
页: [1]
查看完整版本: RTX5中消息邮箱出现的坑